La ricerca ha trovato 106 risultati

da eli9o
27 ago 2008, 00:48
Forum: Combinatoria
Argomento: Dismutazioni!
Risposte: 1
Visite : 2659

Prima di tutto vediamo quanto vale D(n) : applicando il principio di inclusione-esclusione possiamo contare le dismutazioni dell'insieme A: l'idea è di sottrarre a tutte le possibili permutazioni quelle che hanno almeno un punto fisso , poi sommarci quelle che hanno almeno 2 punti fissi, sottrarre q...
da eli9o
22 ago 2008, 13:32
Forum: Combinatoria
Argomento: sicuramente semplice
Risposte: 12
Visite : 4866

Per gli n pari l'idea va bene, il problema affrontandolo così si incontra trattando gli n dispari...
da eli9o
21 ago 2008, 23:19
Forum: Combinatoria
Argomento: problema difficilozzo con estensione...
Risposte: 16
Visite : 7694

Provo a mettere la soluzione per un n generico. I casi possibili sono 6^n e su questo non ci piove... Contare i casi favorevoli significa contare le funzioni surgettive da un insieme di n elementi ad un insieme di 6: il codominio è formato dai 6 possibili risultati e il dominio dagli n lanci. Adesso...
da eli9o
21 ago 2008, 22:34
Forum: Combinatoria
Argomento: sicuramente semplice
Risposte: 12
Visite : 4866

Si potrebbe anche dimostrare che la disuguaglianza stretta vale per ogni $ k $ (ovviamente compreso tra 0 ed n) se si prende $ n>2 $ :roll:
da eli9o
01 ago 2008, 18:38
Forum: Teoria dei Numeri
Argomento: Primi balcanici e facili
Risposte: 10
Visite : 4011

essendo p e q dei primi ho due casi: p=q e r+1=r+5 , che è assurdo; p=r+5 e q=r+1 . Questo sarebbe vero solo se fossero primi anche r+1 ed r+5 altrimenti basta che il loro rapporto sia uguale a quello dei primi (volendo che siano multipli di una stessa costante rispetto ai primi). Ad esempio 3(7+1)...
da eli9o
31 lug 2008, 08:51
Forum: Algebra
Argomento: funzionale che non deve funzionare
Risposte: 9
Visite : 4209

Grazie per le correzioni, alla fine si impara sempre così... eli9o: attento che la negazione di f(n)\leq n \quad\forall n\in \mathbb{N} non e' f(n)> n \quad\forall n\in \mathbb{N} , ma \exists n\in \mathbb{N}: f(n)>n Sì, è vero... Ma diciamo che se non fossi così masochista avrei preso h\in \mathbb ...
da eli9o
30 lug 2008, 23:55
Forum: Algebra
Argomento: funzionale che non deve funzionare
Risposte: 9
Visite : 4209

Premesso che ho iniziato a vedere (e non a fare) funzionali a Cesenatico quest'anno provo a mettere una "soluzione" Supponiamo che f(n) \leq n : allora avremmo f(f(n)) \leq f(n) \leq n ma siccome f(f(n))=n+1987 abbiamo l'assurdo. Quindi abbiamo f(n) > n . Se f(n) non fosse una funzione lin...
da eli9o
30 lug 2008, 14:13
Forum: Teoria dei Numeri
Argomento: coppietta di quadrati... perfetti
Risposte: 9
Visite : 4046

Ok, la mia soluzione era completamente analoga
Avevo postato questo esercizio proprio perchè saltasse fuori quella tecnica (o motto per edriv) quindi dopo una spiegazione così ci si può ritenere profondamente soddisfatti :P

Ciao
da eli9o
29 lug 2008, 20:07
Forum: Ciao a tutti, mi presento:
Argomento: Ciao, mi presento!!
Risposte: 4
Visite : 3302

Se il tuo obbiettivo (non solo quello, per carità) è migliorarti nelle olimpiadi di matematica hai scelto il forum giusto :wink:
Vedrai che con un po' d'allenamento la fase d'istituto diventerà meno insormontabile

Ciao (e benvenuto)
da eli9o
28 lug 2008, 22:05
Forum: Teoria dei Numeri
Argomento: coppietta di quadrati... perfetti
Risposte: 9
Visite : 4046

coppietta di quadrati... perfetti

Determinare tutte le coppie (x,y) di interi positivi tali che x^2+3y e y^2+3x sono simultaneamente quadrati perfetti. Enjoy :P ps: nonostante la soluzione non sia particolarmente lunga io ci ho sbattuto la testa un bel po' prima di riuscirci Edit: avevo sbagliato a scrivere il testo, scusatemi, ora ...
da eli9o
27 lug 2008, 18:41
Forum: Teoria dei Numeri
Argomento: Pseudoprimi
Risposte: 2
Visite : 3140

Usiamo tutti i cannoni possibili e immaginabili, e il bello è che alla fine non ti rispondo nemmeno (ma se non ricordo male sono in pochi a saperti rispondere) ... I numeri di Mersenne sono quei numeri che si possono esprimere come \displaystyle 2^n-1 E' evidente che se \displaystyle n è composto lo...
da eli9o
25 lug 2008, 00:31
Forum: Teoria dei Numeri
Argomento: Galileiana 2006/07, chiarimento.
Risposte: 3
Visite : 2608

c\le\frac{7}{2}k (ovvero <=3, visto che siamo in Z). Secondo me sbagli qua, il fatto che \frac{7}{2} non sia intero non implica che \frac{7}{2}k non sia intero infatti per k pari ottieni i multipli di 7 (che sono valori ottenibili). Presumibilmente i valori "misti" che si possono ottenere...
da eli9o
24 lug 2008, 18:47
Forum: Teoria dei Numeri
Argomento: ancora funzioni aritmetiche!
Risposte: 12
Visite : 4605

Dato che ho una soluzione del 2 "che non fa uso nè di ordini moltiplicativi nè di cose derivanti da essa" la metto. Per l'1 penso di esserci arrivato in fondo ma è una roba lunghissima. Bisogna passare dal fatto che le sommatorie sono moltiplicative? (se non si può dire, vabè capitemi) Ok,...
da eli9o
23 lug 2008, 21:50
Forum: Fisica
Argomento: Non bisogna essere troppo tesi [Halliday]
Risposte: 11
Visite : 8628

Io ho la seconda edizione (la usava mia madre :lol:) comunque anche il risultato del mio libro è $ 40 Kg_p $ ma è diversa la massa del blocco B: a me dice $ 160 kg_p $ che in effetti fa risultare il problema
da eli9o
23 lug 2008, 21:35
Forum: Teoria dei Numeri
Argomento: p|a^2+ab+b^2
Risposte: 20
Visite : 7866

@Skz: mi sembra che il teorema di Dirichlet affermi proprio che se (a,b)=1 allora esistono infiniti primi della forma ax+b Dato che tutto ciò è colpa mia :lol: provo a dare il mio contributo dimostrando che esistono infiniti primi della forma 3k+2 . L'altra non la so ma mi piacerebbe vederla. Suppon...